Howdy, Stranger!

It looks like you're new here. If you want to get involved, click one of these buttons!

How to properly analyze the relationship between the Premises and Conclusions w. Conditionals?!

GUDETAMAGUDETAMA Alum Member
edited November 2019 in Logical Reasoning 114 karma

LR Question Referring To: PT#J07 S#3 Q#17 (Exercising muscles in one back...)

Hey all! Hope all is well, I just have a question about how to properly analyze conditionals in Premises, Sub-Conclusions, and Conclusions? Right now I noticed that my understanding has been pretty lacking and I am not sure if I am steering myself in the right direction or not. If someone could perhaps correct my reasoning or approach that would be very much appreciated.

Premises
Proper Alignment ----> Muscles Opposite Sides Pull Equally

Sub Conclusion
Maintaining a Healthy Back ----> Balanced Muscle Development

Conclusion
Maintaining a Healthy Back ---> Exercise Muscles on Opposite Sides Equally

So upon reading @Sami wonderful explanation here (https://7sage.com/discussion/#/discussion/9561/pt-june-2007-s3-q17-when-excercising-the-muscles) I saw that she identified a gap between the PA and MHB. While I was pouring over the relationship between the two I, perhaps falsely assumed, that Maintaining a Healthy Back ---> Proper Alignment because after considering the Negation of it being Maintaining a Healthy Back and /Proper Alignment I reasoned that Proper Alignment was a Necessary component for Maintaining a Healthy Back...

...and while I am writing this I feel like I am making assumption upon assumption :X because right this very second I am considering whether there are varying degrees of Proper Alignment, what the hell is even this Proper Alignment, what defines Proper Alignment, would 1% of a Improper Alignment be prevent ones back from being thrown into the category of Proper Alignment?! Anyways,

because should /Proper Alignment be the case it seems hard pressed for me to say that one could claim that they are Maintaining a Healthy Back. So upon reasoning or screwing myself over I saw that because Maintaining a Healthy Back ---> Proper Alignment I could connect up the Conditional in the Premises to the Sub Conclusion to get

Premises
- Maintaining a Healthy Back ---> Proper Alignment ---> Muscles Opposite Sides Pull Equally
- Maintaining a Healthy Back ---> Balanced Muscle Development

Conclusion
- Maintaining a Healthy Back (MHB) ---> Exercise Muscles on Opposite Sides Equally

So from the Premises I inferred that Muscles Opposite Sides Pull Equally and Balanced Muscle Development because they were both now connected to Maintaining a Healthy Back. So I guess with that I just equated the two ideas (I am honestly not even sure if you can do that...) and just went off looking for something that would connect either Muscles Opposite Sides Pull Equally or Balanced Muscle Development to Exercise Muscles on Opposite Sides Equally because it seemed to be the only gap remaining.

So then I read @Sami explanation again and under her explanation for Answer Choice: A she says...

" we know that there is a relationship between balanced muscle developed and proper alignment of back, but does the stimulus say that having a balanced muscle development is sufficient/enough to guarantee the proper back alignment? No, its says its needed but does not guarantee that it would lead to a proper back alignment. Other things can also despite having a balanced muscle development could lead someone not to have the proper alignment of back."

And now I am even more lost because I fail to see how the Stimulus says that Balanced Muscle Development is Necessary for Proper Alignment. It seems like all the Premises is doing is just explaining with the conditional (Proper Alignment ----> Muscles Opposite Sides Pull Equally) why the conditional in the Sub Conclusion is the case being (Maintaining a Healthy Back ----> Balanced Muscle Development). If we take what is contained in the Premises to be true and only question the support then wouldn't the already established connection Sami mentioned, Proper Alignment --> Balanced Muscle Development, already connect the Premises to the Sub-Conclusion because it would just be...

Premises
Maintaining a Healthy Back ---> Proper Alignment ----> Muscles Opposite Sides Pull Equally

Conclusion
Maintaining a Healthy Back ---> Exercise Muscles on Opposite Sides Equally

because if we accept the Premises as true cant we chain up the other conditions associated with it which would basically mean that the first gap mentioned by Sami being between Proper Alignment and Maintaining a Healthy Back is essentially nonexistent?

My apologies for the long post I am just concerned with how I feel like I arrived at the correct answer with some fucking bizarre reasoning that I cant even fully comprehend. Any help or clarification would be very much appreciated! Thanks again. I need a damn drink.

Comments

  • SamiSami Live Member Sage 7Sage Tutor
    edited November 2019 10774 karma

    Hey @"MY LEGGGGGGG" ,

    I just want to say bear in mind this is a reply I made three years ago when I first started studying! Do not be mislead by the title. I was neither a Sage nor a Tutor back then -I was a student just trying to be helpful. :joy:

    @"MY LEGGGGGGG" said:
    LR Question Referring To: PT#J07 S#3 Q#17 (Exercising muscles in one back...)

    Hey all! Hope all is well, I just have a question about how to properly analyze conditionals in Premises, Sub-Conclusions, and Conclusions? Right now I noticed that my understanding has been pretty lacking and I am not sure if I am steering myself in the right direction or not. If someone could perhaps correct my reasoning or approach that would be very much appreciated.

    Premises
    Proper Alignment ----> Muscles Opposite Sides Pull Equally

    Sub Conclusion
    Maintaining a Healthy Back ----> Balanced Muscle Development

    Conclusion
    Maintaining a Healthy Back ---> Exercise Muscles on Opposite Sides Equally

    So upon reading @Sami wonderful explanation here (https://7sage.com/discussion/#/discussion/9561/pt-june-2007-s3-q17-when-excercising-the-muscles) I saw that she identified a gap between the PA and MHB. While I was pouring over the relationship between the two I, perhaps falsely assumed, that Maintaining a Healthy Back ---> Proper Alignment

    So you are correct. That's the way the argument is supposed to be read. While not explicitly stated as a premise that MHB--> PA, it is very reasonable to assume that and is therefore not problematic in analysis.
    Instead you want to understand how the argument works.

    Argument#1:

    Premises
    Proper Alignment ----> Muscles Opposite Sides Pull Equally

    Sub Conclusion
    Maintaining a Healthy Back ----> Balanced Muscle Development

    Here, argument is valid only if the chain reads something like this:

    Maintaining a Healthy Back ----> Proper Alignment ----> Muscles Opposite Sides Pull Equally----> Balanced Muscle Development

    Notice the one in bold and Italicized are not explicitly stated as premises in argument. But this is the only way the first sub-conclusion is valid based on that premise. If any part of the chain does not follow, the sub-conclusion of the argument above does not follow.

    As far as the analysis goes, I think like you said it seems reasonable to say that you need proper alignment for maintaining a healthy back. I am not so sure about the second assumption being as reasonable though. Just because something pulls equally does not mean it will result in balanced muscle development.

    Let's take a look at our second argument:

    Sub Conclusion
    Maintaining a Healthy Back ----> Balanced Muscle Development

    Main Conclusion
    Maintaining a Healthy Back ---> Exercise Muscles on Opposite Sides Equally

    Here the argument is valid if Balanced Muscle Development ---> Exercise Muscles on Opposite Sides Equally

    As far as the analysis goes, I again think this is not an awful assumption to make. I can see how if you are exercising muscles and you just workout lets say one arm and not the other, your muscles will develop in one arm and not the other one, you will look funny because you will not have had a balanced muscle development.

    So then I read @Sami explanation again and under her explanation for Answer Choice: A she says...

    " we know that there is a relationship between balanced muscle developed and proper alignment of back, but does the stimulus say that having a balanced muscle development is sufficient/enough to guarantee the proper back alignment? No, its says its needed but does not guarantee that it would lead to a proper back alignment. Other things can also despite having a balanced muscle development could lead someone not to have the proper alignment of back."

    And now I am even more lost because I fail to see how the Stimulus says that Balanced Muscle Development is Necessary for Proper Alignment.

    So remember that chain for argument #1:
    Maintaining a Healthy Back ----> Proper Alignment ----> Muscles Opposite Sides Pull Equally----> Balanced Muscle Development

    Imagine if you had a conditional chain: A-->B-->C--->D
    Wouldn't you say D is necessary for B. Similarly, based on our conditional statement above we can say the Balanced Muscle development is necessary for Proper Alignment. But answer choice "A" is saying Balance Muscle Development is sufficient for Proper Alignment.

    It seems like all the Premises is doing is just explaining with the conditional (Proper Alignment ----> Muscles Opposite Sides Pull Equally) why the conditional in the Sub Conclusion is the case being (Maintaining a Healthy Back ----> Balanced Muscle Development).

    You have listed the premises correctly, but you also have to also understand how the argument works. Necessary Assumption question type is asking you to find a neccessary statement below in the answer choices if the argument is valid. The "necessary" part of the question stem has a sufficient condition. In this case it means that the argument is valid. So what will make your sub-conclusion valid from just that one particular conditional statement? Our chain will ( Maintaining a Healthy Back ----> Proper Alignment ----> Muscles Opposite Sides Pull Equally----> Balanced Muscle Development ).

    I hope this helped.

  • GUDETAMAGUDETAMA Alum Member
    edited November 2019 114 karma

    @Sami Oh my god thank you so so so much!! I think on this part

    Premises
    - Maintaining a Healthy Back ---> Proper Alignment ---> Muscles Opposite Sides Pull Equally
    - Maintaining a Healthy Back ---> Balanced Muscle Development

    I diagrammed and understood as

    • Maintaining a Healthy Back ---> Balanced Muscle Development AND Proper Alignment ---> Muscles Opposite Sides Pull Equally

    so diagramming it on paper with the split and I believed I could somehow infer from my mistaken chaining that Muscles Opposite Sides Pull Equally and Balanced Muscle Development. When in actuality because Maintaining a Healthy Back was determined to be Sufficient for Proper Alignment so its not the case that its a AND relationship that joins the two parts together.

    Edit: I forgot to ask that when I know its not all that great to rely too heavily on Lawgic but in trying to see how these properly link up say we used your example with Proper Alignment --> Balanced Muscle Development in linking up the original argument

    Premises
    Proper Alignment ----> Muscles Opposite Sides Pull Equally

    Sub Conclusion
    Maintaining a Healthy Back ----> Balanced Muscle Development

    Conclusion
    Maintaining a Healthy Back ---> Exercise Muscles on Opposite Sides Equally

    So if we had identified that relationship between the Premises and the Sub Conclusion wouldn't there be many other ways we could possibly map it out that still retains the Sufficiency and Necessity Relationship of the Premises and Sub Conclusion but mean completely different things when put together? Like for example...

    Version #1
    Maintaining a Healthy Back --> Proper Alignment --> Balanced Muscle Development --> Muscles Opposite Sides Pull Equally

    OR

    Version #2
    Proper Alignment --> Muscles Opposite Sides Pull Equally --> Maintaining a Healthy Back --> Balanced Muscle Development

    I am kinda leaning toward Version #2 being wrong because the connection between Proper Alignment and Balanced Muscle development while it can be inferred from the chain that Proper Alignment --> Balanced Muscle Development it is connected in a more indirect way and that in Version #2 it isn't really connecting via Proper Alignment --> Balanced Muscle as that is only inferred and instead the Premises and Sub Conclusion are connected through Muscles Opposite Sides Pull Equally --> Maintaining a Healthy Back which wouldn't be correct.

Sign In or Register to comment.